Last visit was: 25 Apr 2024, 17:10 It is currently 25 Apr 2024, 17:10

Close
GMAT Club Daily Prep
Thank you for using the timer - this advanced tool can estimate your performance and suggest more practice questions. We have subscribed you to Daily Prep Questions via email.

Customized
for You

we will pick new questions that match your level based on your Timer History

Track
Your Progress

every week, we’ll send you an estimated GMAT score based on your performance

Practice
Pays

we will pick new questions that match your level based on your Timer History
Not interested in getting valuable practice questions and articles delivered to your email? No problem, unsubscribe here.
Close
Request Expert Reply
Confirm Cancel
SORT BY:
Date
Math Expert
Joined: 02 Sep 2009
Posts: 92915
Own Kudos [?]: 619029 [29]
Given Kudos: 81595
Send PM
GMAT Club Legend
GMAT Club Legend
Joined: 08 Jul 2010
Status:GMAT/GRE Tutor l Admission Consultant l On-Demand Course creator
Posts: 5960
Own Kudos [?]: 13387 [1]
Given Kudos: 124
Location: India
GMAT: QUANT+DI EXPERT
Schools: IIM (A) ISB '24
GMAT 1: 750 Q51 V41
WE:Education (Education)
Send PM
Manager
Manager
Joined: 12 Mar 2019
Posts: 191
Own Kudos [?]: 104 [0]
Given Kudos: 105
Send PM
Intern
Intern
Joined: 03 Jul 2020
Posts: 3
Own Kudos [?]: 0 [0]
Given Kudos: 1
Send PM
Are both of the integers x and y divisible by 3? (1) x + y and x – y [#permalink]
If X and y are a combination of 3 and 0, then this is not true? 0 is not divisible by 3. Can someone explain why this logic is wrong? Because of this, I picked B.

GMATNinja
Current Student
Joined: 16 Jan 2019
Posts: 631
Own Kudos [?]: 1444 [2]
Given Kudos: 144
Location: India
Concentration: General Management
GMAT 1: 740 Q50 V40
WE:Sales (Other)
Send PM
Re: Are both of the integers x and y divisible by 3? (1) x + y and x – y [#permalink]
2
Kudos
doogyshadow3 wrote:
If X and y are a combination of 3 and 0, then this is not true? 0 is not divisible by 3. Can someone explain why this logic is wrong? Because of this, I picked B.

GMATNinja


0 is divisible by 3. In fact, 0 is divisible by all numbers

In general, if \(\frac{a}{b}=integer\) then \(a\) is divisible by \(b\)

Posted from my mobile device
Director
Director
Joined: 09 Jan 2020
Posts: 966
Own Kudos [?]: 223 [0]
Given Kudos: 434
Location: United States
Send PM
Re: Are both of the integers x and y divisible by 3? (1) x + y and x – y [#permalink]
Bunuel wrote:
Are both of the integers x and y divisible by 3?

(1) x + y and x – y are divisible by 3.
(2) x^2 and y^2 are divisible by 9.


DS20392


(1) If the sum of two numbers and the difference of two numbers are divisible by 3, then each number is divisible by 3. SUFFICIENT.

(2). \(x^2\) and \(y^2\) are divisible by \(3^2\). Therefore x and y are divisible by 3. SUFFICIENT.

Answer is D.
Current Student
Joined: 09 Oct 2020
Posts: 45
Own Kudos [?]: 9 [0]
Given Kudos: 44
Location: India
GMAT 1: 710 Q49 V38
GPA: 4
Send PM
Are both of the integers x and y divisible by 3? (1) x + y and x – y [#permalink]
When we consider Statement 2, why can't x^2 = 18? It just says that it's divisible by 9. So x would not come out to be a number that is divisible by 3. In that case, statement 2 would be insufficient. I ended up picking A as the right choice. Please help!
GMAT Tutor
Joined: 24 Jun 2008
Posts: 4128
Own Kudos [?]: 9244 [0]
Given Kudos: 91
 Q51  V47
Send PM
Re: Are both of the integers x and y divisible by 3? (1) x + y and x – y [#permalink]
Expert Reply
vatsal323 wrote:
When we consider Statement 2, why can't x^2 = 18? It just says that it's divisible by 9. So x would not come out to be a number that is divisible by 3. In that case, statement 2 would be insufficient. I ended up picking A as the right choice. Please help!


If x^2 = 18, then x is not an integer (x would equal 3√2, which is roughly 4.24), but the question tells us x is an integer, so x^2 cannot equal 18.
Current Student
Joined: 09 Oct 2020
Posts: 45
Own Kudos [?]: 9 [0]
Given Kudos: 44
Location: India
GMAT 1: 710 Q49 V38
GPA: 4
Send PM
Are both of the integers x and y divisible by 3? (1) x + y and x – y [#permalink]
IanStewart wrote:
vatsal323 wrote:
When we consider Statement 2, why can't x^2 = 18? It just says that it's divisible by 9. So x would not come out to be a number that is divisible by 3. In that case, statement 2 would be insufficient. I ended up picking A as the right choice. Please help!


If x^2 = 18, then x is not an integer (x would equal 3√2, which is roughly 4.24), but the question tells us x is an integer, so x^2 cannot equal 18.


Oh understood. So basically, while considering either of the statements, one must also assume that everything given as conditions in the question stem itself is the entire sample set?
Manager
Manager
Joined: 16 Dec 2021
Posts: 108
Own Kudos [?]: 15 [0]
Given Kudos: 43
Location: India
GMAT 1: 630 Q45 V31
Send PM
Re: Are both of the integers x and y divisible by 3? (1) x + y and x y [#permalink]
10+17/3 = 27/3, here x and y is not divisible by 3.
17-8/3, here also x and y is not divisible by 3.

So, I thought the rule is x+y can either both be divisible by 3 or both of them not divisible by 3.

Here, the questions says that x+y and x-y is simultaneously divisible by 3. Is this the reason why A is sufficient??
To satisfy both the equation, x and y has to be divisible by 3??

Can some one pls explain here?
Math Expert
Joined: 02 Sep 2009
Posts: 92915
Own Kudos [?]: 619029 [1]
Given Kudos: 81595
Send PM
Re: Are both of the integers x and y divisible by 3? (1) x + y and x y [#permalink]
1
Kudos
Expert Reply
nikitathegreat wrote:
10+17/3 = 27/3, here x and y is not divisible by 3.
17-8/3, here also x and y is not divisible by 3.

So, I thought the rule is x+y can either both be divisible by 3 or both of them not divisible by 3.

Here, the questions says that x+y and x-y is simultaneously divisible by 3. Is this the reason why A is sufficient??
To satisfy both the equation, x and y has to be divisible by 3??

Can some one pls explain here?


In (1), x and y cannot be 10 and 17 because 10 - 17 = -7 is not divisible by 3. Similarly, x and y cannot be 17 and 8 because 17 + 8 = 25 is not divisible by 3. When choosing numbers for x and y, they must satisfy both of the conditions: x + y and x – y must be divisible by 3.

Next, here is why (1) is sufficient. We are given that both x + y and x – y are divisible by 3:
x + y = (a multiple of 3)
x - y = (a multiple of 3)

Sum the above to get 2x = (a multiple of 3). Since the right hand side is a multiple of 3, then the left hand side must also be a multiple of 3, which means that x is a multiple of 3. If x is a multiple of 3, then from x + y = (a multiple of 3) it follows that y must also be a multiple of 3.

Hope it helps.
GMAT Club Bot
Re: Are both of the integers x and y divisible by 3? (1) x + y and x y [#permalink]
Moderator:
Math Expert
92915 posts

Powered by phpBB © phpBB Group | Emoji artwork provided by EmojiOne